Laplace Transform of Heaviside Function

In summary, the conversation discusses determining the Laplace transform of a given function, f(t) = sin(t) for 0 <= t < pi and f(t) = 0 for t >= pi. The discussion includes the use of the Heaviside function, but it is ultimately determined that the basic definition of the Laplace transform should be used. The final solution is found to be e^(-πs) + 1 / (s^2 + 1).
  • #1
_N3WTON_
351
3

Homework Statement


Determine the Laplace transform of the given function:
[itex] f(t) = sin(t) [/itex] for [itex] 0 <= t < \pi [/itex] and [itex] f(t) = 0 [/itex] for [itex] \pi <= t [/itex]

Homework Equations

The Attempt at a Solution


Ok, I've been having some trouble figuring out how I should write the above branched function (sorry for the format, I'm not sure how to write branched function in latex). This is what I have the function as right now:
[itex] f(t) = (1-\sin t)u_{\pi}(t) [/itex]. I can visualize what the graph of the above function should look like, but I sometimes have trouble actually writing it. I was hoping somebody could confirm whether or not this is correct, once I have the function figured out I suspect the rest of the problem probably won't be too difficult.
 
Physics news on Phys.org
  • #2
The question is merely asking you to compute ##L\{f(t)\}## for ##0 \leq t \leq \pi##.

The reason being, the other part of the integral converges to zero. That is:

$$\int_{\pi}^{\infty} f(t) e^{-st} \space dt = \int_{\pi}^{\infty} 0 \space dt = 0$$
 
  • #3
_N3WTON_ said:

Homework Statement


Determine the Laplace transform of the given function:
[itex] f(t) = sin(t) [/itex] for [itex] 0 <= t < \pi [/itex] and [itex] f(t) = 0 [/itex] for [itex] \pi <= t [/itex]

Homework Equations

The Attempt at a Solution


Ok, I've been having some trouble figuring out how I should write the above branched function (sorry for the format, I'm not sure how to write branched function in latex). This is what I have the function as right now:
[itex] f(t) = (1-\sin t)u_{\pi}(t) [/itex]. I can visualize what the graph of the above function should look like, but I sometimes have trouble actually writing it. I was hoping somebody could confirm whether or not this is correct, once I have the function figured out I suspect the rest of the problem probably won't be too difficult.

By trying to exploit fancy methods involving the Heaviside function, etc., you are only going to make a lot of unnecessary work for yourself. It is much more direct to just use the basic definition of the Laplace transform and apply it to your function.

Anyway, in answer to your question: NO, the formula above is incorrect. You can see that for yourself, just by asking yourself what you get for ## 0 < t < \pi ## and for ## t > \pi ##.
 
  • #4
Ray Vickson said:
By trying to exploit fancy methods involving the Heaviside function, etc., you are only going to make a lot of unnecessary work for yourself. It is much more direct to just use the basic definition of the Laplace transform and apply it to your function.
.
I understand, but my issue is I am having trouble seeing what the function is, therefore I do not know what I am taking the Laplace transform of. The reason I arrived at my first solution was because the branched function [itex] f(t) = 1[/itex] for t<c and [itex] f(t) = 0 [/itex] for t>=c is [itex] 1 - u_{c}(t) [/itex] yes? I tried doing something similar with the above problem. I was thinking perhaps [itex] f(t) = sin(t) - u_{\pi}(t) [/itex]?
 
  • #5
_N3WTON_ said:
I understand, but my issue is I am having trouble seeing what the function is, therefore I do not know what I am taking the Laplace transform of. The reason I arrived at my first solution was because the branched function [itex] f(t) = 1[/itex] for t<c and [itex] f(t) = 0 [/itex] for t>=c is [itex] 1 - u_{c}(t) [/itex] yes? I tried doing something similar with the above problem. I was thinking perhaps [itex] f(t) = sin(t) - u_{\pi}(t) [/itex]?

You said ##f(t) = \sin(t)## if ##0 \leq t < \pi##, and ## f(t) = 0## if ##t \geq \pi##. I honestly cannot see what possible problem you can have interpreting and plotting that. You are told exactly what is ##f(t)## for ## t \in [0,\pi) ## and you are told exactly what is ##f(t)## for ##t \geq \pi##.

Of course, when taking the LT we ignore the part ##t < 0##; alternatively, you could take ##f(t) = 0## for ##t < 0##.
 
  • #6
Ray Vickson said:
You said ##f(t) = \sin(t)## if ##0 \leq t < \pi##, and ## f(t) = 0## if ##t \geq \pi##. I honestly cannot see what possible problem you can have interpreting and plotting that. You are told exactly what is ##f(t)## for ## t \in [0,\pi) ## and you are told exactly what is ##f(t)## for ##t \geq \pi##.

Of course, when taking the LT we ignore the part ##t < 0##; alternatively, you could take ##f(t) = 0## for ##t < 0##.
I'm not having a problem interpreting/plotting the function, what I am having a problem with is writing it using the unit step function...
 
  • #7
Edit: I see now how stupid my questions have been, sorry :/ lol
 
Last edited:
  • #8
Anyhow, here is the solution I have so far:
[itex] \mathcal{L}(f(t))= \int_{0}^{\infty} e^{-st}f(t)\,dt [/itex]
[itex] = \int_{0}^{\pi} e^{-st}\sin t\,dt + \int_{\pi}^{\infty} e^{-st}(0)\,dt[/itex]
[itex] \frac{e^{-st}}{s^{2}+1}[-s\sin t + \cos t]\bigr|_{0}^{\pi} [/itex]
[itex] (\frac{e^{-st}}{s^{2}+1}[-s\sin \pi + \cos \pi]) - (\frac{1}{s^{2}+1}[-s \sin 0 + \cos 0]) [/itex]
[itex] = \frac{e^{-\pi s}+1}{s^{2}+1} [/itex]
 
Last edited:
  • #9
_N3WTON_ said:
Anyhow, here is the solution I have so far:
[itex] \mathcal{L}(f(t))= \int_{0}^{\infty} e^{-st}f(t)\,dt [/itex]
[itex] = \int_{0}^{\pi} e^{-st}\sin t\,dt + \int_{\pi}^{\infty} e^{-st}(0)\,dt[/itex]
[itex] \frac{e^{-st}}{s^{2}+1}[-s\sin t + \cos t]\bigr|_{0}^{\pi} [/itex]
[itex] (\frac{e^{-st}}{s^{2}+1}[-s\sin \pi + \cos \pi]) - (\frac{1}{s^{2}+1}[-s \sin 0 + \cos 0]) [/itex]
[itex] = \frac{e^{-\pi s}+1}{s^{2}+1} [/itex]

I have not checked all the details, but it looks OK at a glance.
 
  • #10
Ray Vickson said:
I have not checked all the details, but it looks OK at a glance.
great, thanks for the help sorry if I was being difficult earlier, I think I was getting hung up on a topic that didn't really even apply to this specific problem...
 
  • #11
_N3WTON_ said:
I understand, but my issue is I am having trouble seeing what the function is, therefore I do not know what I am taking the Laplace transform of. The reason I arrived at my first solution was because the branched function [itex] f(t) = 1[/itex] for t<c and [itex] f(t) = 0 [/itex] for t>=c is [itex] 1 - u_{c}(t) [/itex] yes? I tried doing something similar with the above problem. I was thinking perhaps [itex] f(t) = sin(t) - u_{\pi}(t) [/itex]?
If you want to do it this way, you want to multiply ##\sin t## by ##[1-u_\pi(t)]##. The factor ##[1-u_\pi(t)]## picks off only the first half-cycle of ##\sin t##.
 
  • #12
vela said:
If you want to do it this way, you want to multiply ##\sin t## by ##[1-u_\pi(t)]##. The factor ##[1-u_\pi(t)]## picks off only the first half-cycle of ##\sin t##.
Thank you, I originally wanted to do it that way just because I wanted some practice writing branched functions in terms of the unit step function..
 

1. What is the Laplace Transform of Heaviside Function?

The Laplace Transform of Heaviside Function, also known as the unit step function, is a mathematical function that represents a constant value of 0 for negative input values and a constant value of 1 for positive input values. It is commonly denoted as u(t) or H(t).

2. What is the purpose of using the Laplace Transform of Heaviside Function?

The Laplace Transform of Heaviside Function is useful in solving differential equations as it converts a time-domain function into a frequency-domain function. This makes it easier to analyze and solve complex systems.

3. How is the Laplace Transform of Heaviside Function calculated?

The Laplace Transform of Heaviside Function can be calculated using the integral formula: L{u(t)} = ∫0 e-st dt = 1/s, where s is the complex frequency variable.

4. What is the inverse Laplace Transform of Heaviside Function?

The inverse Laplace Transform of Heaviside Function is given by the formula: L-1{1/s} = u(t), which means that when the Laplace Transform is applied to the Heaviside Function, it results in the function itself.

5. How is the Laplace Transform of Heaviside Function used in real-world applications?

The Laplace Transform of Heaviside Function has various applications in fields such as engineering, physics, and mathematics. It is used to model and analyze systems with switching behavior, such as electrical circuits, control systems, and mechanical systems. It is also used in signal processing and communication systems.

Similar threads

  • Calculus and Beyond Homework Help
Replies
3
Views
1K
  • Calculus and Beyond Homework Help
Replies
2
Views
1K
  • Calculus and Beyond Homework Help
Replies
10
Views
1K
  • Calculus and Beyond Homework Help
Replies
2
Views
991
  • Calculus and Beyond Homework Help
Replies
3
Views
886
  • Calculus and Beyond Homework Help
Replies
1
Views
59
  • Calculus and Beyond Homework Help
Replies
6
Views
2K
  • Calculus and Beyond Homework Help
Replies
1
Views
776
  • Calculus and Beyond Homework Help
Replies
6
Views
357
  • Calculus and Beyond Homework Help
Replies
4
Views
2K
Back
Top